User Avatar
trankathy2294
Joined
Apr 2025
Subscription
Free
User Avatar
trankathy2294
Friday, May 30 2014

Hi David Michael, thanks for your excellent response! I really appreciate the detailed message.

I definitely see the first sentence of the stimulus as "A ----> B" [salespeople ---> steer customers toward products from which they make their highest commision]

So, all salespeople in major health stores (let us call them C) help us reach

C ---> A ---> B

(salespeople in major health stores ---> salespeople ---> steer customers . . .)

So there is no flaw. But IF it had said, 'Salespeople generally tend to steer customers toward products from which they make highest commissions' and from that conclude that the salespeople in the health stores do the same, would that be a whole to part flaw? Even then, I'm not sure it would because the main conclusion is strictly concerned about the quality of products being inaccurate.

User Avatar

Thursday, May 29 2014

trankathy2294

PT69.S4.Q16 - salespeople always steer customers

http://classic.7sage.com/lsat_explanations/lsat-69-section-4-question-16/

Hey test-takers! If you have PT69, care to discuss this question with me?

Upon review, the flaw is glaring and answer choice (B) makes sense.

During the timed test, I chose (C) and now I still can't see why the author did not make that error. H/she went from "salespeople" (whole) to "salespeople in major health stores" (part). This was the reasoning used to the reach the conclusion, so why is it not right to attack it?

Thank you!

User Avatar
trankathy2294
Thursday, May 29 2014

Thank you so much!

User Avatar

Saturday, May 24 2014

trankathy2294

Logic Chain (ish) question

Hello,

I've got a common question about linking up a logic chain. I just re-watched the video lesson but it's not precisely the answer to my question. Any help is appreciated!

If you have:

Premise: A ---> B ----> C

-------------------------------------

Conclusion: D ---> C

Is it correct to try to make D ---> A and D ---> B (either one works, along with their contrapositives) OR would it be correct to make A ---> D and B ---> D?

My question is from PT 47 section 3, number 21. According to the video, it is the former method, but I remember doing it the other way sometimes. I got the Q right doing it in my head but it'd be great to confirm it. Thanks again

PrepTests ·
PT140.S4.P3.Q15
User Avatar
trankathy2294
Thursday, May 22 2014

For the explanatory role in Question 15, I said to myself, OK - the author drew an inference about the explanatory role of intensive training (i.e., how intensive training explains) superior performance. What is this inference? Intensive training depends on interest and desire, which is precisely discussed in the final paragraph.

I guess it is connected with the motivational factors too, but the inferences are allowed to be related! I don't believe they have to be mutually exclusive.

User Avatar
trankathy2294
Monday, May 19 2014

That's awesome! I don't recall PT 44 LG on the top of my head, but going from -10 to -5 is good. Keep doing em! It's the only way to get better. Cheers

User Avatar
trankathy2294
Sunday, May 18 2014

Hello,

I just did PT 65 yesterday. I actually had an opposite experience. I found the first LR section of that test harder than the second section. I also found the second LR particularly easier than other PTs. Perhaps the difference is due to our different strengths in question types. But what we can be sure is that tests usually have 1 hard and 1 easy LR. Also, this is the first time I did this, but I marked down quickly how much time I had left when I reached certain questions. For the 2nd LR section, by the time I reached question 13, I had 19 mins left. By the time I reached question 20, I had 9.5 minutes left. I ended up having to guess 1 question; I ended up with -3. (First LR I had -6 too). (I'm not sure if telling you this helps but I just wanted to respond to your post on PT 65)

The RC was ridiculously hard, IMO.

User Avatar
trankathy2294
Saturday, May 17 2014

You can try to make a games bundle for yourself. For example, you can pick out some basic linear/sequencing games, use a paper clip to hold them together. Then the grouping in/out ones. Then double layering sequencing. You get the idea, SIMILAR game types, bundle them together. Then do them so you can start to see the patterns for each game type because the inferences and the rules are so often recycled! It helps you build confidence, too. I think that this should help you slightly better than reviewing game sections after game sections.

I don't think you need to be good at math. I can't think of any game where I need to have knowledge of math (like, algebra.)

Lastly, this works for me and a lot of other test takers I'm sure. Give it a try: recycle your work. So do the questions that say 'IF this goes in THIS SLOT, what could be true?' first. Then later, if there's a question that asks you for 'What must be false' you can look BACK at your work in the other question and quickly eliminate answer choices. Also, spend time doing the set-up diagram. I know it's hard because you want to move onto the questions but it pays off well later.

User Avatar
trankathy2294
Sunday, Jun 08 2014

Thanks so much for the explanation, YesManJr!

@

In case anyone wants to practice with this rule in the Logical Reasoning section, PT63, S3, Q. 22 is excellent practice!

User Avatar

Saturday, Jun 07 2014

trankathy2294

LG Rule with 'otherwise'

Hello!

Simple question + looking for confirmation.

If a rule says: If Sarah is cool, she will go to the party with her sister; otherwise, she will not.

Sarah cool ---> Party with sister

~ Sarah Cool ---> ~ Party with sister

Yes? I read somewhere that this is like an 'if and only if' rule... can someone comment?

Their contrapositives:

~Party with sister ---> ~Sarah cool

Party with sister --> Sarah cool

User Avatar
trankathy2294
Saturday, May 03 2014

Gotcha- thank you Jonathan!

PrepTests ·
PT134.S2.Q21
User Avatar
trankathy2294
Thursday, Jan 02 2014

Hi J.Y., in answer choice (B), would it have to add something along the lines of "....making the comparison between the 2 texts unreliable" in order for us to say that it successfully attacks the supporting premise? Thanks

User Avatar
trankathy2294
Friday, May 02 2014

Hi Jonathan, thanks for your input.

(Warning: partial PT 60 question SPOILER alert below)

I re-read the conclusions for both questions. In the PT 60 example, I can see that the conclusion is supported by the surveys... but it's a terrible support because the economists make a different claim (they made a claim about the strongest incentive, NOT preference). In the medical condition video lesson, the conclusion is also supported by the studies... but there isn't another claim to compare it to.

Still quite lost. So what I'm getting is that there's no black/white rule about using another survey/study to weaken answer choices and that it depends on the context of the argument?

User Avatar
trankathy2294
Friday, May 02 2014

Hi there! Thanks for responding. Yes- you are right, answer choice (a) in the medical example does indeed weaken. I am not confused about that particular question but rather, why in that case, 'another study' is acceptable as a weakener, yet in the other example I mentioned, 'another study' is out of scope.

I hope I clarified it better!

User Avatar

Friday, May 02 2014

trankathy2294

Weaken question - trap answer choice or no?

Hi fellow 7sagers,

As a review tonight I decided to write down steps I take for weakening questions. I’d appreciate any response to this question I have... PLEASE! It’s been bothering me for the past 2 hours. I realize that this might require going to the videos that I refer to below – so I thank you in advance for your time spent!

For Weaken Qs, we are taught to attack the premise-conclusion relationship; that is, the support for our conclusion. So I tried to come up with an example:

If the Premise/Conclusion is: TV sales increase, because Survey A indicates so.

A trap answer choice would be ‘Survey B indicates otherwise’ (right?) because sure, Survey B is a contradiction, but our premise about Survey A still holds true and we can’t doubt its validity since it was given to us.

This is corroborated by a video explanation I watched, PT 60 Section 1 Question 13 (in short – there’s an answer choice (B) saying Survey X says some dangerously out of scope stuff – but our premise is about Survey Y saying whatever to support the conclusion.)

BUT I was watching the “Serious Medical Condition – Weaken Question” video lesson and in that question, Answer Choice (A) serves as a perfectly acceptable weaken-er! But it is another one of those ‘in another study....this was shown...” !!

So does that type of answer choice weaken the argument? Help appreciated!! Thank you :)

User Avatar
trankathy2294
Sunday, Jun 01 2014

Gotcha! Thanks A LOT again for typing all that out. This is great, learning something new everyday. So generally = usually = cannot apply that property to the group 100% of the time. I have not seen this often on the PTs but if I see it, I will make a note to be careful about distinguishing the flaws. I assume that the answer choice will mirror that language though, by saying that the author cannot presume without justification that what one group generally does together is what each individual group would generally do by him/herself.

Nice reference to the wood-table example! That question really bugged me because all the answer choices seemed the same (It was metal and a desk though).

Confirm action

Are you sure?